Finding three Constants given only relative Max, and y intercept

Click For Summary
To find the constants a, b, and c for the quadratic function f(x) = ax² + bx + c, the given conditions are a relative maximum at (5, 12) and a y-intercept at (0, 3). The y-intercept provides c = 3. The derivative f'(x) = 2ax + b must equal zero at the maximum point, leading to the equation 10a + b = 0, which simplifies to b = -10a. By substituting (5, 12) into the function, the equation 12 = 75a + 3 is derived, resulting in a = 0.12 and b = -1.2. The solution process highlights the importance of using both the function and its derivative to find the constants.
Hypnos_16
Messages
148
Reaction score
1

Homework Statement



Find constants a, b, and c such that the graph of f(x) = ax2 + bx + c has a relative maximum at (5 , 12) and crosses the the y - axis at (0 , 3).

Homework Equations



Not sure what to use

The Attempt at a Solution



-- i don't have one --
 
Physics news on Phys.org
Since you're given the y-intercept (0, 3), you know that when x = 0, f(x) = 3. Plug in these values into
f(x) = ax2 + bx + c
and you should be able to find one of the 3 constants.

You were given the hint in your other thread that the derivative at an extrenum is 0. Find the derivative of
f(x) = ax2 + bx + c
and plug in some values. Plug in some more values into f(x) and you'll find yourself with 2 equations and 2 unknowns.
 
so after plugging in (0,3) i got that c = 3.
f'(x) = 2ax + b
after plugging in (5,12) being the max i got that

f'(5) = 2(a)(5) + b
f'(5) = 10a + b

but i don't see where i can really go from there.
 
Hypnos_16 said:
so after plugging in (0,3) i got that c = 3.
f'(x) = 2ax + b
after plugging in (5,12) being the max i got that

f'(5) = 2(a)(5) + b
f'(5) = 10a + b

but i don't see where i can really go from there.

You missed this:
eumyang said:
You were given the hint in your other thread that the derivative at an extrenum is 0.
So
f'(5) = 10a + b = 0.

You also need to plug in (5, 12) into
f(x) = ax2 + bx + c
and you'll have two equations with two unknowns.
 
So since i know that (5,12) i know that when x is 5 y is 12
soooooo
f'(x) = 2ax + b
f'(x) = 10a + b = 0
b = 10a

f(5) = 25a + 5b + 3
f(5) = 25a + 5(10a) + 3
f(5) = 25a + 50a + 3
f(5) = 75a + 3
12 = 75a + 3
9 = 75a
a = 0.12
?

something like that?
then just fill in what i know to find b

b = 10a
b = 1.2
?
 
That's the idea, but you have one sign error...

Hypnos_16 said:
So since i know that (5,12) i know that when x is 5 y is 12
soooooo
f'(x) = 2ax + b
f'(x) = 10a + b = 0
b = 10a
Here. Should be b = -10a.
 
Question: A clock's minute hand has length 4 and its hour hand has length 3. What is the distance between the tips at the moment when it is increasing most rapidly?(Putnam Exam Question) Answer: Making assumption that both the hands moves at constant angular velocities, the answer is ## \sqrt{7} .## But don't you think this assumption is somewhat doubtful and wrong?

Similar threads

  • · Replies 4 ·
Replies
4
Views
2K
  • · Replies 10 ·
Replies
10
Views
3K
Replies
18
Views
4K
  • · Replies 5 ·
Replies
5
Views
1K
Replies
3
Views
2K
  • · Replies 24 ·
Replies
24
Views
2K
  • · Replies 10 ·
Replies
10
Views
3K
  • · Replies 6 ·
Replies
6
Views
2K
  • · Replies 18 ·
Replies
18
Views
3K
Replies
1
Views
2K